13 events
when toggle format what by license comment
Dec 2 at 21:10 history became hot network question
Dec 2 at 13:00 comment added Carl-Fredrik Nyberg Brodda @Gro-Tsen I suppose it is worth mentioning that if $G$ is finitely generated then every proper subgroup is indeed contained in a maximal subgroup (proved using Zorn’s Lemma as usual).
Dec 2 at 12:10 comment added YCor @Gro-Tsen the Prüfer group $C_{p^\infty}$ (increasing union of cyclic groups $C_{p^n}$) has no maximal proper subgroup. Not so magic.
Dec 2 at 12:01 history edited semisimpleton CC BY-SA 4.0
added 166 characters in body
Dec 2 at 10:58 comment added Gro-Tsen @YCor “Not every proper subgroup is contained in a maximal one.” 😱 HOW? WHY? WHAT IS THIS MAGIC? 😵 I feel like I've just been told that Santa Claus doesn't exist. (And yes, indeed, when you think of it, it's obvious, but it makes you reconsider why maximal ideals exist.) This is one for the big list.
Dec 2 at 10:27 history edited semisimpleton CC BY-SA 4.0
added 166 characters in body
Dec 2 at 10:24 comment added semisimpleton Oh, I see! It's strange that Zorn's Lemma can be used to prove that "every proper ideal is contained in a maximal ideal", but it can't be used to prove that "every proper subgroup is contained a maximal subgroup".
Dec 2 at 9:04 comment added YCor The group $C_{p^\infty}\times C_6$ has exactly 2 maximal proper subgroups but is not cyclic. (In your reasoning "any element in $G-M_1\cup M_2$ generates the whole group" is not true — indeed not every proper subgroup is contained in a maximal one.)
Dec 2 at 7:55 comment added YCor The group $C_{p^\infty}\times C_6$ has exactly 2 maximal subgroups but is not cyclic. (In your reasoning "any element in $G-M_1\cup M_2$ generates a cyclic subgroup" is not true.)
Dec 2 at 5:17 comment added Carl-Fredrik Nyberg Brodda (My comment should obviously read "any non-cyclic group")
Dec 2 at 4:04 answer added Jeremy Rouse timeline score: 6
Dec 2 at 3:48 comment added Carl-Fredrik Nyberg Brodda Experimenting with GAP suggests that any group with exactly $3$ maximal subgroups is a $2$-group (and there are many such $2$-groups). Is there some simple argument for why this is true (or a counterexample)?
Dec 2 at 3:28 history asked semisimpleton CC BY-SA 4.0